How can we determine $2^kappa$ for singular $kappa$ assuming that $2^lambda=lambda^+$ whenever...












1














I got an exercise in set theory and can't seem to solve it:
If we assume that $2^lambda = lambda^+ $ holds for every singular cardinal with $2^{operatorname{cof}(lambda)}<lambda$, then how can we determine the value of $2^kappa$ from the value of $2^{<kappa}$ for all singular cardinals $kappa$?
I tried using Silver's Theorem, but couldn't seem to get it right...










share|cite|improve this question









New contributor




Jan T. is a new contributor to this site. Take care in asking for clarification, commenting, and answering.
Check out our Code of Conduct.

























    1














    I got an exercise in set theory and can't seem to solve it:
    If we assume that $2^lambda = lambda^+ $ holds for every singular cardinal with $2^{operatorname{cof}(lambda)}<lambda$, then how can we determine the value of $2^kappa$ from the value of $2^{<kappa}$ for all singular cardinals $kappa$?
    I tried using Silver's Theorem, but couldn't seem to get it right...










    share|cite|improve this question









    New contributor




    Jan T. is a new contributor to this site. Take care in asking for clarification, commenting, and answering.
    Check out our Code of Conduct.























      1












      1








      1







      I got an exercise in set theory and can't seem to solve it:
      If we assume that $2^lambda = lambda^+ $ holds for every singular cardinal with $2^{operatorname{cof}(lambda)}<lambda$, then how can we determine the value of $2^kappa$ from the value of $2^{<kappa}$ for all singular cardinals $kappa$?
      I tried using Silver's Theorem, but couldn't seem to get it right...










      share|cite|improve this question









      New contributor




      Jan T. is a new contributor to this site. Take care in asking for clarification, commenting, and answering.
      Check out our Code of Conduct.











      I got an exercise in set theory and can't seem to solve it:
      If we assume that $2^lambda = lambda^+ $ holds for every singular cardinal with $2^{operatorname{cof}(lambda)}<lambda$, then how can we determine the value of $2^kappa$ from the value of $2^{<kappa}$ for all singular cardinals $kappa$?
      I tried using Silver's Theorem, but couldn't seem to get it right...







      set-theory cardinals






      share|cite|improve this question









      New contributor




      Jan T. is a new contributor to this site. Take care in asking for clarification, commenting, and answering.
      Check out our Code of Conduct.











      share|cite|improve this question









      New contributor




      Jan T. is a new contributor to this site. Take care in asking for clarification, commenting, and answering.
      Check out our Code of Conduct.









      share|cite|improve this question




      share|cite|improve this question








      edited Dec 26 '18 at 22:43









      Andrés E. Caicedo

      64.7k8158246




      64.7k8158246






      New contributor




      Jan T. is a new contributor to this site. Take care in asking for clarification, commenting, and answering.
      Check out our Code of Conduct.









      asked Dec 26 '18 at 18:36









      Jan T.

      61




      61




      New contributor




      Jan T. is a new contributor to this site. Take care in asking for clarification, commenting, and answering.
      Check out our Code of Conduct.





      New contributor





      Jan T. is a new contributor to this site. Take care in asking for clarification, commenting, and answering.
      Check out our Code of Conduct.






      Jan T. is a new contributor to this site. Take care in asking for clarification, commenting, and answering.
      Check out our Code of Conduct.






















          1 Answer
          1






          active

          oldest

          votes


















          2














          Let me consider separately two cases, and afterward indicate how to tell the cases apart.



          Case 1: $2^mu$ has the same value for all sufficiently large $mu<kappa$. In this case, we can invoke the Bukovsky-Hechler theorem (see for example Corollary 5.17 in http://fa.its.tudelft.nl/~hart/onderwijs/set_theory/Jech/05-AC_and_cardinal_arithmetic.pdf ) to conclude that $2^kappa=2^{<kappa}$.



          Case 2: As $mu$ ranges over infinite cardinals $<kappa$, the exponential $2^mu$ increases cofinally often. So the supremum $2^{<kappa}$ of these exponentials is a cardinal $lambda$ with cf$(lambda)=,$cf$(kappa)<kappa$. So $2^{text{cf}(lambda)}$ is one of the cardinals whose supremum defines $lambda$ and is, by the case hypothesis, $<lambda$. Now the hypothesis of your exercise gives us that $2^lambda=lambda^+$. So we have
          $$
          lambda=2^{<kappa}leq2^kappaleq2^lambda=lambda^+,
          $$

          which means $2^kappa$ has one of just two possible values, namely $lambda$ and $lambda^+$. But the first of these isn't really possible, by König's theorem, because it has cofinality $<kappa$. So $2^kappa=lambda^+=(2^{<kappa})^+$.



          To complete the answer, I still need to tell you how to distinguish the two cases, given only $2^{<kappa}$. Fortunately, I already pointed out that in Case 2, $2^{<kappa}$ (there called $lambda$) has cofinality $<kappa$. In Case 1, on the other hand, $2^{<kappa}$ is equal to $2^mu$ for arbitrarily large $mu<kappa$ and therefore, by König's theorem again, cannot have cofinality $<kappa$.



          So the final result is (under the hypothesis of the exercise) that $2^kappa$ equals $2^{<kappa}$ if this has cofinality $geqkappa$, and equals $(2^{<kappa})^+$ otherwise. (Less rigorous but easy to remember summary: $2^kappa$ has the smallest value it could possibly have, given $2^{<kappa}$ and given König's theorem.)






          share|cite|improve this answer





















            Your Answer





            StackExchange.ifUsing("editor", function () {
            return StackExchange.using("mathjaxEditing", function () {
            StackExchange.MarkdownEditor.creationCallbacks.add(function (editor, postfix) {
            StackExchange.mathjaxEditing.prepareWmdForMathJax(editor, postfix, [["$", "$"], ["\\(","\\)"]]);
            });
            });
            }, "mathjax-editing");

            StackExchange.ready(function() {
            var channelOptions = {
            tags: "".split(" "),
            id: "69"
            };
            initTagRenderer("".split(" "), "".split(" "), channelOptions);

            StackExchange.using("externalEditor", function() {
            // Have to fire editor after snippets, if snippets enabled
            if (StackExchange.settings.snippets.snippetsEnabled) {
            StackExchange.using("snippets", function() {
            createEditor();
            });
            }
            else {
            createEditor();
            }
            });

            function createEditor() {
            StackExchange.prepareEditor({
            heartbeatType: 'answer',
            autoActivateHeartbeat: false,
            convertImagesToLinks: true,
            noModals: true,
            showLowRepImageUploadWarning: true,
            reputationToPostImages: 10,
            bindNavPrevention: true,
            postfix: "",
            imageUploader: {
            brandingHtml: "Powered by u003ca class="icon-imgur-white" href="https://imgur.com/"u003eu003c/au003e",
            contentPolicyHtml: "User contributions licensed under u003ca href="https://creativecommons.org/licenses/by-sa/3.0/"u003ecc by-sa 3.0 with attribution requiredu003c/au003e u003ca href="https://stackoverflow.com/legal/content-policy"u003e(content policy)u003c/au003e",
            allowUrls: true
            },
            noCode: true, onDemand: true,
            discardSelector: ".discard-answer"
            ,immediatelyShowMarkdownHelp:true
            });


            }
            });






            Jan T. is a new contributor. Be nice, and check out our Code of Conduct.










            draft saved

            draft discarded


















            StackExchange.ready(
            function () {
            StackExchange.openid.initPostLogin('.new-post-login', 'https%3a%2f%2fmath.stackexchange.com%2fquestions%2f3053197%2fhow-can-we-determine-2-kappa-for-singular-kappa-assuming-that-2-lambda%23new-answer', 'question_page');
            }
            );

            Post as a guest















            Required, but never shown

























            1 Answer
            1






            active

            oldest

            votes








            1 Answer
            1






            active

            oldest

            votes









            active

            oldest

            votes






            active

            oldest

            votes









            2














            Let me consider separately two cases, and afterward indicate how to tell the cases apart.



            Case 1: $2^mu$ has the same value for all sufficiently large $mu<kappa$. In this case, we can invoke the Bukovsky-Hechler theorem (see for example Corollary 5.17 in http://fa.its.tudelft.nl/~hart/onderwijs/set_theory/Jech/05-AC_and_cardinal_arithmetic.pdf ) to conclude that $2^kappa=2^{<kappa}$.



            Case 2: As $mu$ ranges over infinite cardinals $<kappa$, the exponential $2^mu$ increases cofinally often. So the supremum $2^{<kappa}$ of these exponentials is a cardinal $lambda$ with cf$(lambda)=,$cf$(kappa)<kappa$. So $2^{text{cf}(lambda)}$ is one of the cardinals whose supremum defines $lambda$ and is, by the case hypothesis, $<lambda$. Now the hypothesis of your exercise gives us that $2^lambda=lambda^+$. So we have
            $$
            lambda=2^{<kappa}leq2^kappaleq2^lambda=lambda^+,
            $$

            which means $2^kappa$ has one of just two possible values, namely $lambda$ and $lambda^+$. But the first of these isn't really possible, by König's theorem, because it has cofinality $<kappa$. So $2^kappa=lambda^+=(2^{<kappa})^+$.



            To complete the answer, I still need to tell you how to distinguish the two cases, given only $2^{<kappa}$. Fortunately, I already pointed out that in Case 2, $2^{<kappa}$ (there called $lambda$) has cofinality $<kappa$. In Case 1, on the other hand, $2^{<kappa}$ is equal to $2^mu$ for arbitrarily large $mu<kappa$ and therefore, by König's theorem again, cannot have cofinality $<kappa$.



            So the final result is (under the hypothesis of the exercise) that $2^kappa$ equals $2^{<kappa}$ if this has cofinality $geqkappa$, and equals $(2^{<kappa})^+$ otherwise. (Less rigorous but easy to remember summary: $2^kappa$ has the smallest value it could possibly have, given $2^{<kappa}$ and given König's theorem.)






            share|cite|improve this answer


























              2














              Let me consider separately two cases, and afterward indicate how to tell the cases apart.



              Case 1: $2^mu$ has the same value for all sufficiently large $mu<kappa$. In this case, we can invoke the Bukovsky-Hechler theorem (see for example Corollary 5.17 in http://fa.its.tudelft.nl/~hart/onderwijs/set_theory/Jech/05-AC_and_cardinal_arithmetic.pdf ) to conclude that $2^kappa=2^{<kappa}$.



              Case 2: As $mu$ ranges over infinite cardinals $<kappa$, the exponential $2^mu$ increases cofinally often. So the supremum $2^{<kappa}$ of these exponentials is a cardinal $lambda$ with cf$(lambda)=,$cf$(kappa)<kappa$. So $2^{text{cf}(lambda)}$ is one of the cardinals whose supremum defines $lambda$ and is, by the case hypothesis, $<lambda$. Now the hypothesis of your exercise gives us that $2^lambda=lambda^+$. So we have
              $$
              lambda=2^{<kappa}leq2^kappaleq2^lambda=lambda^+,
              $$

              which means $2^kappa$ has one of just two possible values, namely $lambda$ and $lambda^+$. But the first of these isn't really possible, by König's theorem, because it has cofinality $<kappa$. So $2^kappa=lambda^+=(2^{<kappa})^+$.



              To complete the answer, I still need to tell you how to distinguish the two cases, given only $2^{<kappa}$. Fortunately, I already pointed out that in Case 2, $2^{<kappa}$ (there called $lambda$) has cofinality $<kappa$. In Case 1, on the other hand, $2^{<kappa}$ is equal to $2^mu$ for arbitrarily large $mu<kappa$ and therefore, by König's theorem again, cannot have cofinality $<kappa$.



              So the final result is (under the hypothesis of the exercise) that $2^kappa$ equals $2^{<kappa}$ if this has cofinality $geqkappa$, and equals $(2^{<kappa})^+$ otherwise. (Less rigorous but easy to remember summary: $2^kappa$ has the smallest value it could possibly have, given $2^{<kappa}$ and given König's theorem.)






              share|cite|improve this answer
























                2












                2








                2






                Let me consider separately two cases, and afterward indicate how to tell the cases apart.



                Case 1: $2^mu$ has the same value for all sufficiently large $mu<kappa$. In this case, we can invoke the Bukovsky-Hechler theorem (see for example Corollary 5.17 in http://fa.its.tudelft.nl/~hart/onderwijs/set_theory/Jech/05-AC_and_cardinal_arithmetic.pdf ) to conclude that $2^kappa=2^{<kappa}$.



                Case 2: As $mu$ ranges over infinite cardinals $<kappa$, the exponential $2^mu$ increases cofinally often. So the supremum $2^{<kappa}$ of these exponentials is a cardinal $lambda$ with cf$(lambda)=,$cf$(kappa)<kappa$. So $2^{text{cf}(lambda)}$ is one of the cardinals whose supremum defines $lambda$ and is, by the case hypothesis, $<lambda$. Now the hypothesis of your exercise gives us that $2^lambda=lambda^+$. So we have
                $$
                lambda=2^{<kappa}leq2^kappaleq2^lambda=lambda^+,
                $$

                which means $2^kappa$ has one of just two possible values, namely $lambda$ and $lambda^+$. But the first of these isn't really possible, by König's theorem, because it has cofinality $<kappa$. So $2^kappa=lambda^+=(2^{<kappa})^+$.



                To complete the answer, I still need to tell you how to distinguish the two cases, given only $2^{<kappa}$. Fortunately, I already pointed out that in Case 2, $2^{<kappa}$ (there called $lambda$) has cofinality $<kappa$. In Case 1, on the other hand, $2^{<kappa}$ is equal to $2^mu$ for arbitrarily large $mu<kappa$ and therefore, by König's theorem again, cannot have cofinality $<kappa$.



                So the final result is (under the hypothesis of the exercise) that $2^kappa$ equals $2^{<kappa}$ if this has cofinality $geqkappa$, and equals $(2^{<kappa})^+$ otherwise. (Less rigorous but easy to remember summary: $2^kappa$ has the smallest value it could possibly have, given $2^{<kappa}$ and given König's theorem.)






                share|cite|improve this answer












                Let me consider separately two cases, and afterward indicate how to tell the cases apart.



                Case 1: $2^mu$ has the same value for all sufficiently large $mu<kappa$. In this case, we can invoke the Bukovsky-Hechler theorem (see for example Corollary 5.17 in http://fa.its.tudelft.nl/~hart/onderwijs/set_theory/Jech/05-AC_and_cardinal_arithmetic.pdf ) to conclude that $2^kappa=2^{<kappa}$.



                Case 2: As $mu$ ranges over infinite cardinals $<kappa$, the exponential $2^mu$ increases cofinally often. So the supremum $2^{<kappa}$ of these exponentials is a cardinal $lambda$ with cf$(lambda)=,$cf$(kappa)<kappa$. So $2^{text{cf}(lambda)}$ is one of the cardinals whose supremum defines $lambda$ and is, by the case hypothesis, $<lambda$. Now the hypothesis of your exercise gives us that $2^lambda=lambda^+$. So we have
                $$
                lambda=2^{<kappa}leq2^kappaleq2^lambda=lambda^+,
                $$

                which means $2^kappa$ has one of just two possible values, namely $lambda$ and $lambda^+$. But the first of these isn't really possible, by König's theorem, because it has cofinality $<kappa$. So $2^kappa=lambda^+=(2^{<kappa})^+$.



                To complete the answer, I still need to tell you how to distinguish the two cases, given only $2^{<kappa}$. Fortunately, I already pointed out that in Case 2, $2^{<kappa}$ (there called $lambda$) has cofinality $<kappa$. In Case 1, on the other hand, $2^{<kappa}$ is equal to $2^mu$ for arbitrarily large $mu<kappa$ and therefore, by König's theorem again, cannot have cofinality $<kappa$.



                So the final result is (under the hypothesis of the exercise) that $2^kappa$ equals $2^{<kappa}$ if this has cofinality $geqkappa$, and equals $(2^{<kappa})^+$ otherwise. (Less rigorous but easy to remember summary: $2^kappa$ has the smallest value it could possibly have, given $2^{<kappa}$ and given König's theorem.)







                share|cite|improve this answer












                share|cite|improve this answer



                share|cite|improve this answer










                answered Dec 27 '18 at 2:31









                Andreas Blass

                49.3k351106




                49.3k351106






















                    Jan T. is a new contributor. Be nice, and check out our Code of Conduct.










                    draft saved

                    draft discarded


















                    Jan T. is a new contributor. Be nice, and check out our Code of Conduct.













                    Jan T. is a new contributor. Be nice, and check out our Code of Conduct.












                    Jan T. is a new contributor. Be nice, and check out our Code of Conduct.
















                    Thanks for contributing an answer to Mathematics Stack Exchange!


                    • Please be sure to answer the question. Provide details and share your research!

                    But avoid



                    • Asking for help, clarification, or responding to other answers.

                    • Making statements based on opinion; back them up with references or personal experience.


                    Use MathJax to format equations. MathJax reference.


                    To learn more, see our tips on writing great answers.





                    Some of your past answers have not been well-received, and you're in danger of being blocked from answering.


                    Please pay close attention to the following guidance:


                    • Please be sure to answer the question. Provide details and share your research!

                    But avoid



                    • Asking for help, clarification, or responding to other answers.

                    • Making statements based on opinion; back them up with references or personal experience.


                    To learn more, see our tips on writing great answers.




                    draft saved


                    draft discarded














                    StackExchange.ready(
                    function () {
                    StackExchange.openid.initPostLogin('.new-post-login', 'https%3a%2f%2fmath.stackexchange.com%2fquestions%2f3053197%2fhow-can-we-determine-2-kappa-for-singular-kappa-assuming-that-2-lambda%23new-answer', 'question_page');
                    }
                    );

                    Post as a guest















                    Required, but never shown





















































                    Required, but never shown














                    Required, but never shown












                    Required, but never shown







                    Required, but never shown

































                    Required, but never shown














                    Required, but never shown












                    Required, but never shown







                    Required, but never shown







                    Popular posts from this blog

                    Human spaceflight

                    Can not write log (Is /dev/pts mounted?) - openpty in Ubuntu-on-Windows?

                    File:DeusFollowingSea.jpg